You are on page 1of 26

ECON 104: Mathematics for Economics

SMU School of Economics; Fall 2020


Homework Assignments
Instructor: Takashi Kunimoto
November 11, 2020

You are supposed to upload your work (in pdf file) at “Assignments” at our
class website (on eLearn) by 5:00pm on the date specified below.

1 Homework 1 (Due Date: Aug 25 (Tue), 2020)


Question 1.1 (24 points) In each of the following implications, where x, y,
and z are real numbers, decide: (i) if the implication is true; and (ii) if the
converse implication is true.

1. x = 4 ⇒ x = 2.

2. (x = 2 and y = 5) ⇒ x + y = 7.

3. (x − 1)(x − 2)(x − 3) = 0 ⇒ x = 1.

4. x2 + y 2 = 0 ⇒ x = 0 or y = 0.

5. (x = 0 and y = 0) ⇒ x2 + y 2 = 0.

6. xy = xz ⇒ y = z.

Question 1.2 (20 points) Determine which of the following formulas are
true. If any formula is false, find a counter-example to demonstrate this.

1. A ⊆ B ⇔ A ∪ B = B.

2. A ⊆ B ⇔ A ∩ B = A.

3. A ∩ B = A ∩ C ⇒ B = C.

1
4. A ∪ B = A ∪ C ⇒ B = C.

5. A = B ⇔ (x ∈ A ⇔ x ∈ B).

Question 1.3 (10 points) Let f (x) = x/(1+x2 ). Answer the following ques-
tions.
√ √
1. Compute f (−1/10), f (0), f (1/ 2), f ( π), and f (2).

2. Show that f (−x) = −f (x) for all x and that f (1/x) = f (x) for x 6= 0.

Question 1.4 (10 points) Let f (x) = 3x2 + 2x − 1. Answer the following
questions.

1. Show that for all h 6= 0,

f (x + h) − f (x)
= 6x + 2 + 3h,
h
0
and use this result to find f (x).
0 0 0
2. Find the value of f (0), f (−2), and f (3).

Question 1.5 (12 points) A firm’s profit function is π(q) = 24q − q 2 − 5,


where q denotes the quantity of the good supplied. Find the marginal profit,
0
i.e., π (q), and the value of q which maximizes profits.

Question 1.6 (24 points) Find the derivatives of the following:

1. x6 .

2. 3x11 .

3. x50 .

4. −4x−7 .
x12
5. 12
.
−2
6. x2
.
3
7. 3 x.

−2
8. √ .
x x

2
2 Homework 2 (Due Date: Sep 1 (Tue), 2020)
Question 2.1 (15 points) Differentiate the following functions.

1. f (x) = 8x4 + 2 x.

2. f (x) = x−1 (x2 + 1) x.

3. f (x) = √1 .
x3

x−2
4. f (x) = √
x+1
.
x2 +x+1
5. f (x) = x2 −x+1
.

Question 2.2 (8 points) Use the chain rule to find dy/dx for the following:

1. y = 5u4 , where u = 1 + x2 .

2. y = u − u6 , where u = 1 + 1/x.

Question 2.3 (15 points) Compute the following limits.

1. limx→2 (x2 + 3x − 5).


1
2. limy→−3 y+8
.
x3 −2x−1
3. limx→0 x5 −x2 −1
.
x3 +3x2 −2x
4. limx→0 x
.
(x+h)3 −x3
5. limh→0 h
, where h 6= 0.

Question 2.4 (12 points) Compute the second derivatives of the following:

1. y = x5 − 3x4 + 2

2. y = x.

3. y = (1 + x2 )10 .

4. y = 1 + x2 .

Question 2.5 (16 points) Find the first and second derivatives of the fol-
lowing:

1. y = e−3x

3
3
2. y = 2ex

3. y = e1/x
2 −3x+1
4. y = 5e2x

Question 2.6 (9 points) Determine the domains of the following functions:

1. y = ln(x + 1)

2. y = ln 3x−1

1−x

3. y = ln |x|

Question 2.7 (9 points) Find the intervals where the following functions are
nondecreasing:

1. y = (ln x)2 − 4

2. y = ln(ex + e−x )

3. y = x − 32 ln(x2 + 2)

Question 2.8 (16 points) Consider the following macroeconomic model:

1. Y = C + I

2. C = f (Y − T )

3. T = α + βY ,

where Y is GDP; C is consumption, T denotes taxes, and α and β are con-


0
stants. Assume that f ∈ (0, 1) and β ∈ (0, 1). Answer the following questions.

1. From the three equations above, derive the equation Y = f ((1 − β)Y −
α) + I.

2. Differentiate the equation Y = f ((1 − β)Y − α) + I with respect to I and


find an expression for dY /dI.

3. Examine the sign of dY /dI.

4
3 Homework 3 (Due Date: Sep 8 (Tue), 2020)
Question 3.1 (16 points) The function f is defined over all x ∈ [−2, 2] by
the formula
1 √
f (x) = x3 4 − x2 .
3
Answer the following questions.

1. Find the intervals where f non-decreases, and the intervals where f non-
increases.

2. Sketch the graph of f .



3. Explain why f has an inverse g on [0, 3].
0 √  √
4. Find g 13 3 . (Hint: f (1) = 13 3).

Question 3.2 (15 points) Let f (x) = (1 + x)m , where m is any real number.
Answer the following questions.

1. Prove that f (x) ≈ 1 + mx for x close to 0.

2. Use the formula above, find approximations to the following numbers:



3 1 1/3

(a) 1.1 = 1 + 10

5 1 1/5

(b) 33 = 2 1 + 32
√ √
(c) 3 9 = 3 8 + 1
(d) (0.98)25 .

Question 3.3 (7 points) For what value of a is the following function con-
tinuous for all x? 
ax − 1 if x ≤ 1
f (x) =
3x2 + 1 if x > 1

Question 3.4 (12 points) Evaluate the following limits:

1. limx→0+ (x2 + 3x − 4)
x+|x|
2. limx→0− x

x+|x|
3. limx→0+ x
−1
4. limx→0+ √
x

5
x
5. limx→3+ x−3
x
6. limx→3− x−3

Question 3.5 (12 points) Find all first- and second-order partial derivatives
(including cross derivatives) for the following functions:

1. z = 3x + 4y

2. z = x3 y 2

3. z = x5 − 3x2 y + y 6

4. z = x/y

5. z = (x − y)/(x + y)
p
6. z = x2 + y 2
0 0
Question 3.6 (7 points) Prove that if z = (ax + by)2 , then xzx + yzy = 2z,
where a, b are constants.

Question 3.7 (7 points) Show that x2 − y 2 = c is a level curve of f (x, y) =


2 2
ex e−y + x4 − 2x2 y 2 + y 4 for all values of the constant c.

Question 3.8 (8 points) Find the distances between the following pairs of
points.

1. (−1, 2, 3) and (4, −2, 0)

2. (a, b, c) and (a + 1, b + 1, c + 1)
0 0
Question 3.9 (6 points) Let F (x, y, z) = x2 exz +y 3 exy . Calculate Fx (1, 1, 1), Fy (1, 1, 1),
0
and Fz (1, 1, 1).

Question 3.10 (10 points) Let

g(x, y, z) = 2x2 − 4xy + 10y 2 + z 2 − 4x − 28y − z + 24

for all (x, y, z) ∈ R3 . Answer the following questions.

1. Calculate g(2, 1, 1), g(3, −4, 2), and g(1, 1, a + h) − g(1, 1, a).

2. Find all partial derivatives of the first and second orders (including cross-
derivatives).

6
4 Homework 4 (Due Date: Sep 15 (Tue), 2020)
Question 4.1 (8 points) For each of the following equations below, answer
the question: if y = f (x) is a differentiable function that satisfies the equation,
0
what is y ? Here a is a positive constant.

1. x2 + y 2 = a2
√ √ √
2. x + y = a

3. x4 − y 4 = x2 y 3

4. exy − x2 y = 1

Question 4.2 (8 points) In the following cases, find dz/dt by using the chain
rule.
Let z = F (x, y).

1. F (x, y) = x + y 2 ; x = t2 ; y = t3

2. F (x, y) = xp y q , x = at, y = bt where p, q, a, b are constants.


√ √
Question 4.3 (8 points) Let Y = 10KL − K − L. Assume that K =
0.2t + 5 and L = 5et/10 . Find dY /dt when t = 0.

Question 4.4 (8 points) Use the chain rule to find ∂w/∂t for the following
cases.

1. w = xy 2 z 3 , where x = t2 , y = s, and z = t.

2. w = x2 + y 2 + z 2 , where x = t + s, y = ets , and z = s3 .

Question 4.5 (8 points) The equation 3x2 − 3xy 2 + y 3 + 3y 2 = 4 defines y


implicitly as a function h(x) of x in a neighborhood of the point (1, 1). Find
0
h (1).

Question 4.6 (10 points) Let

g(µ, ε) = [(1 + µ)(1 + ε)α ]1/(1−β) − 1,

where α and β are constants. Show that if µ and ε are close to 0, then

(µ + αε)
g(µ, ε) ≈ .
1−β

7
Question 4.7 (6 points) Calculate the differentials of the following func-
tions.

1. z = x3 + y 3
2
2. z = xey
3. z = ln(x2 − y 2 )
0 0 00
Question 4.8 (8 points) Find zx , zy , and zxy when x3 + y 3 + z 3 − 3z = 0.

Question 4.9 (6 points) Let


   
0 1 −1 1 −1 5
A= and B = .
2 3 7 0 1 9
Compute A + B, A − B, and 5A − 3B.

Question 4.10 (8 points) Compute the products AB and BA, if possible,


when A and B are given as follows:

1.    
0 −2 −1 4
A= and B =
3 1 1 5
2.  
  2 −2
8 3 −2
A= and B =  4 3 
1 0 4
1 −5
3.  
  3 1
−1 0
A= and B =  −1 1 
2 4
0 2
4.  
0 
A =  −2  and B = 0 −2 3
4

Question 4.11 (6 points) We say that a square matrix A is idempotent if


A2 = A. Show that the matrix
 
2 −2 −4
 −1 3 4 
1 −2 −3
is idempotent.

8
Question 4.12 (8 points) Consider the general 2 × 2 matrix
 
a b
A= .
c d
Answer the following questions.

1. Prove that A2 = (a + d)A − (ad − bc)I2 .


2. Find an example of a 2 × 2 matrix A such that A2 = 0 but A 6= 0.

Question 4.13 (8 points) An n × n matrix P is said to be orthogonal if


0
P P = In . Answer the following questions.

1. For λ = ±1/ 2, show that the matrix
 
λ 0 λ
P =  λ 0 −λ 
0 1 0
is orthogonal.
2. Show that the 2 × 2 matrix
 
p −q
q p

is orthogonal if and only if p2 + q 2 = 1.

5 Homework 5 (Due Date: Sep 22 (Tue), 2020)


Question 5.1 (8 points) Use Cramer’s rule to solve the following system of
equations.

1.

x1 − x2 = 0
x1 + 3x2 + 2x3 = 0
x1 + 2x2 + x3 = 0

2.

x + 3y − 2z = 1
3x − 2y + 5z = 14
2x − 5y + 3z = 1

9
Question 5.2 (9 points) Define the matrix
 
1 t 0
At =  −2 −2 −1  ,
0 1 t

where t is a real number. Answer the following questions.

1. Calculate the determinant of At .

2. Confirm that |At | =


6 0.

3. Show that for a certain value of t, one has A3t = I3 .

Question 5.3 (12 points) Evaluate the determinant of each of the following
matrices as simply as possible:

1.  
3 0 1
A =  1 0 −1 
2 0 5

2.  
1 2 3 4
 0 −1 2 4 
B= 
 0 0 3 −1 
−3 −6 −9 −12

3.  
a1 − x a2 a3 a4
 0 −x 0 0 
C= ,
 0 1 −x 0 
0 0 1 −x
where a1 , a2 , a3 , a4 and x are real numbers.

Question 5.4 (7 points) For what values of x is the inner product of (x, x −
1, 3) and (x, x, 3x) equal to 0?

Question 5.5 (10 points) Let a = (1, 2, 1) and b = (−3, 0, −2). Answer the
following questions.

1. Find numbers x1 and x2 such that x1 a + x2 b = (5, 4, 4).

2. Prove that there are no real numbers x1 and x2 satisfying x1 a + x2 b =


(−3, 6, 1).

10
Question 5.6 (8 points) The line L is given by (x1 , x2 , x3 ) = (−t + 2, 2t −
1, t + 3) where t ∈ R. Answer the following questions.

1. Verify that the point a = (2, −1, 3) lies on L.

2. Verify that the point (1, 1, 1) does not lie on L.

3. Find the equation for the plane passing through a that is orthogonal to
L.

4. Find the point P where L intersects the plane 3x1 + 5x2 − x3 = 6.

Question 5.7 (12 points) Calculate the inverses of the following matrices,
if they exist.

1.  
2 3
A= .
4 5
2.  
1 0 2
B =  2 −1 0 
0 2 −1
3.  
1 0 0
C =  −3 −2 1 
4 −16 8

Question 5.8 (10 points) Let


   
1 0 t 1 0 0
At =  2 1 t  and B =  0 0 1  .
0 1 1 0 1 0

Answer the following questions.

1. For what values of t does At have an inverse?

2. Find a matrix X such that B + XA−1 −1


1 = A1 .

Question 5.9 (12 points) An equilibrium model of labor demand and output
pricing leads to the following system of equations:
0
pF (L) − w = 0
pF (L) − wL − B = 0.

11
Here, F (·) is a production satisfying the following properties: F (L) > 0 for
0 00
all L > 0 and F is twice differentiable with F (L) > 0 and F (L) < 0. All
the variables are positive. Regard w and B as exogenous, so that p and L are
endogenous variables which are functions of w and B. Answer the following
questions.
1. Find expressions for ∂p/∂w, ∂p/∂B, ∂L/∂w, and ∂L/∂B.
2. What can be said about the signs of these partial derivatives?
3. Show that ∂L/∂w < 0.
Question 5.10 (12 points) The following system of equations defines u =
u(x, y) and v = v(x, y) as differentiable functions of x and y around the point
P = (x, y, u, v) = (1, 1, 1, 2):
uα + v β = 2β x + y 3
uα v β − v β = x − y,
where α and β are positive constants. Answer the following questions.
1. Differentiate the system, and find ∂u/∂x, ∂u/∂y, ∂v/∂x, and ∂v/∂y at
the point P .
2. Find an approximation to u(0.99, 1.01).

6 Homework 6 (No Need for Submission)


0
Question 6.1 Suppose that f (x, y) is homogeneous of degree 2, with fx (2, 3) =
0
4 and fy (4, 6) = 12. Find f (6, 9).

Question 6.2 Evaluate the following limits. You are required to be clear about
the whole process of how you come to your evaluation for each limit.
1.
x−4
lim
x→∞ x2 + 1

2. r
2 + 5x
lim
x→−∞ x−1
3.
(ax + b)2
lim ,
x→∞ (a − x)(b − x)

where a, b are some constants.

12
Question 6.3 The following system of equations defines both u = u(x, y) and
v = v(x, y) as twice continuously differentiable functions of x and y around the
point P where (x, y, u, v) = (−1, 1, 1, 2):
xu3 + v = y 2
3uv − x = 7.
Answer the following questions.
1. Find the differentials of u and v expressed in terms of the differentials of
x and y.
2. Use Cramer’s rule to find ∂u/∂x and ∂v/∂x at P .
3. Use Cramer’s rule to find ∂u/∂y and ∂v/∂y at P .
4. If x is decreased by 0.01 and y is increased by 0.02 from their values at
P , what are the new values of u and v, approximately?
Question 6.4 Let f be a single variable function defined as follows:
 
1 1+x
f (x) = ln .
2 1−x
Answer the following questions.
1. What is the domain of f ?
2. Show that f is a strictly increasing function over the domain you found
in the previous question.
3. Show that the range of f is (−∞, ∞).
4. Argue why f has an inverse function g.
5. Derive the inverse function g explicitly.
0
6. Find g 12 ln 2 .


Question 6.5 Let  


a11 a12 a13
A =  a21 a22 a23 
a31 a32 a33
be a 3 × 3 matrix where aij is a constant for each i, j = 1, 2, 3. We note that
|A| denotes the determinant of a matrix A and Aij denotes the cofactor corre-
sponding to (i, j) component of the matrix A. (Hint: if a matrix B contains two
rows or columns which are proportional to each other (for example, identical),
then |B| = 0.) Answer the following questions.

13
1. What are A21 , A22 , and A23 ?

2. Show that

a11 a12 a13

a11 a12 a13 = a11 A21 + a12 A22 + a13 A23 = 0.

a31 a32 a33

3. Show that

a11 a12 a13

a21 a22 a23 = a11 A31 + a12 A32 + a13 A33 = 0.

a11 a12 a13

4. Show that a21 A11 +a22 A12 +a23 A13 = 0 and a21 A31 +a22 A32 +a23 A33 = 0.

5. Show that a31 A11 +a32 A12 +a33 A13 = 0 and a31 A21 +a32 A22 +a33 A23 = 0.

6. Compute the product of the following two matrices and simplify its ex-
pression.   
a11 a12 a13 A11 A21 A31
 a21 a22 a23   A12 A22 A32  .
a31 a32 a33 A13 A23 A33

7. Assume that |A| = 6 0. Using all the previous results you obtain thus far,
derive explicitly the formula of the inverse matrix of A (denoted by A−1 ).

Question 6.6 Find the (global) maximum or minimum points for the following
functions.

1.
8
f (x) =
3x2+4
2.
f (x) = 5(x + 2)4 − 3

3.
1
f (x) =
1 + x4
for x ∈ [−1, 1]

4.
−2
f (x) =
2 + x2

14
5. √
f (x) = 2 − 1−x

6.
2
f (x) = 100 − e−x

Question 6.7 Find possible extreme (i.e., local maximum or minimum) points
for g(x) = x3 ln x, for x ∈ (0, ∞).

7 Homework 7 (Due Date: Oct 13 (Tue), 2020)


Question 7.1 (12 points) Answer the following questions regarding a single
firm’s behavior.

1. A firm produces Q = 2 L units of a commodity when L units of labor
are employed. If the price obtained per unit is $160, and the price per
unit of labor is $40, what value of L maximizes profits π(L)?

2. A firm produces Q = f (L) units of a commodity when L units of labor are


0 00
employed. Assume that f (L) > 0 and f (L) < 0. If the price obtained
per unit is 1 and price per unit of labor is w, what is the first-order
condition for maximizing profits at L = L∗ ?

3. By implicitly differentiating the first-order condition in the previous ques-


tion with respect to w, find how L∗ changes when w change.

Question 7.2 (15 points) Consider the monopolist who faces the inverse de-
mand function P (Q) = a − Q. Assume that k is the cost of producing one unit
of output and 0 < k < a. Answer the following questions.

1. Find the profit maximizing output Q∗ and the associated monopoly profit
π(Q∗ ).

2. How does the monopoly profit react to changes in k? (Hint: Find


dπ(Q∗ )/dk)

3. The government argues that the monopoly produces too little. It wants
to induce the monopolist to produce Q̂ = a − k units by granting a
subsidy s per unit of output. Calculate the subsidy s required to reach
the target.

Question 7.3 (10 points) Let f (x) = 4x2 − 40x + 80 for x ∈ [0, 8]. Answer
the following questions

15
1. Draw the graph of f (x).

2. Find the maximum and minimum of f (x) over [0, 8].

Question 7.4 (10 points) Consider the function f defined for all x ∈ [−1, 1]
by 
x if x ∈ (−1, 1)
f (x) =
0 if x = −1 or x = 1
Answer the following questions.

1. Is this function continuous? If so, verify it. If not, argue why.

2. Does the function f attain maximum or minimum? If so, provide them


explicitly. If not, argue why?

Question 7.5 (8 points) Consider the function f defined for all x ∈ R by


f (x) = x3 − 12x. Find the stationary points of f , and classify them by using
both the first- and second-order conditions.

Question 7.6 (12 points) Decide where the following functions are convex
and determine possible inflection points:

1.
x
f (x) =
1 + x2
2.
1−x
f (x) =
1+x
3.
f (x) = xex

Question 7.7 (12 points) Let


1 1
f (x) = ln(x + 1) − x + x2 − x3 .
2 6
Answer the following questions.

1. Find the domain of the function f .

2. Prove that for x in the domain,

0 x2 − x3
f (x) =
2(x + 1)

16
3. Find all possible extreme points and inflection points.

4. Check f (x) as x → (−1)+ and sketch the graph on the interval (−1, 2].

Question 7.8 (9 points) Consider a quadratic form: Q(x, y) = ax2 + 2xy+
cy 2 (a 6= 0). Answer the following questions.

1. Show that if Q(x, y) is positive definite, then a > 0 and c > 0.

2. Suppose that a > 0 and ac = 1/2. What is the definiteness?

3. Suppose that c = −1. Find the range of value a in which Q(x, y) is


negative definite.

Question 7.9 (12 points) A quadratic form Q(x, y) = ax2 + 2bxy + cy 2 is


written by using a symmetric matrix A, where
 
a b
A= .
b c

Answer the following questions.

1. Show that if Q(x, y) is positive definite, then the diagonal elements of A


are positive.

2. Show that if Q(x, y) is negative definite, then the diagonal elements of A


are negative.

3. Show that if Q(x, y) is negative definite, then it is negative semidefinite.

8 Homework 8 (Due Date: Oct 20 (Tue), 2020)


Question 8.1 (10 points) Let f : R2 → R be a function by f (x, y) = x2 +
y 2 − 6x + 8y + 35. Answer the following questions.

1. Show that the function f has a minimum point. Find it.

2. Show that f (x, y) can be written in the form f (x, y) = (x − 3)2 + (y +


4)2 +10. Explain why this shows that you have really found the minimum
in the previous question.

Question 8.2 (14 points) The profit function of a firm is given as π(x, y) =
px + qy − αx2 − βy 2 , where p and q are the prices per unit and αx2 + βy 2 are
the costs of producing and selling x units of the first good and y units of the
other. And α and β are positive constants. Answer the following questions.

17
1. Find the values of x and y that maximizes profits. Denote them by x∗
and y ∗ . Verify that the second-order conditions are satisfied.
2. Define π ∗ (p, q) = π(x∗ , y ∗ ). Verify that ∂π ∗ (p, q)/∂p = x∗ and ∂π ∗ (p, q)/∂q =
y∗.
Question 8.3 (20 points) Let f (x, y) = xe−x (y 2 −4y). Answer the following
questions.
1. Find all stationary points of f .
2. Classify all the stationary points by using the second-order conditions.
3. Let S = {(x, y) ∈ R2 | 0 ≤ x ≤ 5, 0 ≤ y ≤ 4}. Prove that f has global
maximum and minimum points in S and find them.
4. Find the tangent line to the level curve xe−x (y 2 − 4) = e − 4 at the point
where x = 1 and y = 4 − e.
2
Question 8.4 (9 points) Let f (x) = e−x and F (u) = ln u. Verify that f (x)
and F (f (x)) achieve the maximum at the same values of x.

Question 8.5 (20 points) A firm produces and sells a product in two sepa-
rate markets. When the price in market 1 is p per unit, and the price in market
2 is q per unit, the demands in the two markets are, respectively
Q1 = a − bp, Q2 = c − dq.
The cost function is C(Q1 , Q2 ) = α + β(Q1 + Q2 ), and all constants a, b, c, d, α,
and β are positive. Answer the following questions.
1. Find the firm’s profit π as a function of the prices p and q.
2. Find the pair (p∗ , q ∗ ) that maximizes profits.
3. Suppose it becomes unlawful to discriminate by price, so that the firm
must charge the same price in the two markets. What price p̂ will now
maximize profits?
4. In the case of β = 0, find the firm’s loss of profit if it has to charge
the same price in both markets. Keep in mind that you are required to
obtain the simplest possible expression for this.
Question 8.6 (12 points) Use the Lagrangian method to obtain the solution
candidate for the following problem:
min −40x + x2 − 2xy − 20y + y 2 subject to x + y = 15.
(x,y)∈R2

18
Question 8.7 (15 points) Consider the following minimization problem:

min (x + 2)2 + y 2 subject to y 2 − x(x + 1)2 = 0.


x,y

Answer the following questions.

1. Explicitly show that the Lagrangian method does not work.

2. Find the solution to the minimization problem. When answering this


question, you are required to clarify your argument for this.

3. Explain why the Lagrangian method does not work.

9 Homework 9 (Due Date: Oct 27 (Tue), 2020)


Question 9.1 (16 points) A single-product firm intends to produce Q units
of output as cheaply √ as possible. By using K units of capital and L units of
labor, it can produce K + L units of output. Suppose the prices of capital and
labor are, respectively, r and w. Answer the following questions.

1. Assume that Q > w/2r, where r, w, and Q are positive constants. Solve
the firm’s problem:

min rK + wL subject to K + L = Q.
K,L

2. Let K ∗ and L∗ be the solution to the constrained optimization problem.


Define C(r, w, Q) = rK ∗ + wL∗ . Show that ∂C/∂Q corresponds to the
Lagrange multiplier.

Question 9.2 (20 points) Consider the consumer demand problem

max α ln(x − a) + β ln(y − b) subject to px + qy = m,


x≥0,y≥0

where α, β, a, b, p, q, and m are positive constants, with α + β = 1 and m >


ap + bq. Answer the following questions.

1. Show that if (x∗ , y ∗ ) solves the constrained optimization problem above,


then expenditure on the two goods is given by the two linear functions:

px∗ = αm + pa − α(pa + qb)


qy ∗ = βm + qb − β(pa + qb)

19
2. Let U ∗ (p, q, m) = U (x∗ , y ∗ ). Show that ∂U ∗ /∂m > 0 and
∂U ∗ ∂U ∗ ∗ ∂U ∗ ∂U ∗ ∗
=− x and =− y .
∂p ∂m ∂q ∂m
Question 9.3 (14 points) Solve the problem
max x2 + y 2 − 2x + 1 subject to x2 + 4y 2 = 16
(x,y)∈R2

Question 9.4 (14 points) Solve the problem


min ln(2 + x2 ) + y 2 subject to x2 + 2y = 2
(x,y)∈R2

Note that you should take it for granted that the minimum value exists.

Question 9.5 (16 points) Let U (x, y) = xa + y a , where a ∈ (0, 1). Answer
the following questions.
1. Prove that U (x, y) is concave when x > 0 and y > 0.
2. Solve the following problem:
max U (x, y) subject to px + qy = m,
x≥0,y≥0

where p, q, and m are positive constants.


Question 9.6 (20 points) Suppose that U (x, y) denotes the utility enjoyed
by a person when having x hours of leisure per day (24 hours) and y units per
day of other goods. The person gets an hourly wage of w and pays an average
price of p of the other goods, so that
py = w(24 − x), (∗)
assuming that the person spends all that is earned. Answer the following ques-
tions.
1. Show that maximizing U (x, y) subject to the constraint (∗) leads to the
equation
0 0
pU1 (x, y) = wU2 (x, y) (∗∗)
2. Suppose that the equations (∗) and (∗∗) define x and y as differentiable
functions x(p, w) and y(p, w) of p and w. Show that, with “appropriate”
conditions on U (x, y),
00 00 0
∂x (24 − x)(wU22 − pU12 ) + pU2
= 00 00 00 .
∂w p2 U11 − 2pwU12 + w2 U22
Note that you must be explicit about the appropriate conditions you
need.

20
10 Homework 10 (Due Date: Nov 3 (Tue),
2020)
Question 10.1 (18 points) Consider the following problem:

max 2 f (x, y) = x + y subject to g(x, y) = x2 + y = 1.


(x,y)∈R

Answer the following questions.

1. Find the solution to the constrained maximization problem.

2. Explain the solution geometrically by drawing appropriately level curves


for f (x, y) together with the graph of of the parabola x2 + y = 1.

3. Replace the constraint by x2 + y = 1.1 and solve the problem in this case.
Find the corresponding change in the optimal value of f (x, y) = x + y,
and check to see if this change is approximately equal to λ · 0.1.

Question 10.2 (22 points) Solve the problem

max{x + 4y + z} subject to x2 + y 2 + z 2 = 216 and x + 2y + 3z = 0

Question 10.3 (30 points) Consider the following utility maximization prob-
lem:
max x + ln(1 + y) subject to px + y ≤ m, x ≥ 0, y ≥ 0,
x,y

where p and m are positive constants. Answer the following questions.

1. Write down the necessary Kuhn-Tucker conditions (with non-negativity


constraints) for point (x∗ , y ∗ ) to be a solution.

2. Find the solution to the problem, for all positive values of p and m.

Question 10.4 (30 points) Consider the following maximization problem:

max xy + x + y subject to x2 + y 2 ≤ 2 and x + y ≤ 1


(x,y)∈R2

Answer the following questions.

1. Draw the two constraints in the x − y plane.

2. Write down the Kuhn-Tucker conditions for this problem.

3. Find the solution to the constrained maximization problem.

21
11 Homework 11 (Due Date: Nov 10 (Tue),
2020)
Question 11.1 (20 points) Consider the following maximization problem:
2
max f (x, y) = 2 − (x − 1)2 − ey subject to x2 + y 2 ≤ a,
x,y

where a is a positive constant. Answer the following questions.

1. Write down the Kuhn-Tucker conditions for the solution of the problem.

2. Find the solution “candidates” that satisfy the Kuhn-Tucker conditions.


Note that you may need to distinguish the case of different values of a
for this. The same comment applies to the rest of the questions.

3. Find the solution to the problem.

4. Let (x∗ , y ∗ ) be a solution to the maximization problem. Define f ∗ (a) =


f (x∗ , y ∗ ) as the optimal value of f , which depends on a. Verify that
df ∗ (a)/da is equivalent to the Lagrange multiplier derived from the Kuhn-
Tucker conditions.
R1 R1
Question 11.2 (10 points) Suppose 0 (f (x)−2g(x))dx = 6 and 0 (2f (x)+
R1
2g(x))dx = 9. Then, find I = 0 (f (x) − g(x))dx.

Question 11.3 (15 points) Use integration by parts to evaluate the follow-
ing:

1. Z 4 √
t ln(t)dt
1

2. Z 2
(x − 2)e−x/2 dx
0

3. Z 3
(3 − x)3x dx
0

Question 11.4 (15 points) Find the following integrals by integration by sub-
stitution.

22
1. Z
(x2 + 1)8 2xdx

2. Z
(x + 2)10 dx

3.
2x − 1
Z
dx
x2 −x+8
Question 11.5 (24 points) Calculate the following integrals.
Z 1
2
(1) (2x + 1)6 dx,
Z0 3

(2) x 3 − x dx,
Z0 2
xdx
(3) ,
x2 + 1
Z0 1
(4) xex dx,
Z0 2
(5) ln x dx,
1
Z 1
(6) (x − 1)2 e−x dx.
0

Question 11.6 (16 points) Determine the following integrals, if they con-
verge. Indicate those that diverge.
1. Z ∞
1
dx
1 x3
2. Z ∞
1
√ dx
1 x
3. Z 0
ex dx
−∞

4. Z a
x
√ dx,
0 a2 − x 2
where a > x.

23
12 Homework 12 (No Need for Submission)
Question 12.1 A sum of $12,000 is invested at 4% annual interest. Answer
the following questions.

1. What will this amount have grown to after 15 years?

2. How much should you have deposited in a bank account five years ago
in order to have $50,000 today, given that the interest rate has been 5%
per year over the period?

Question 12.2 Examine the convergence or divergence of the following series:

1. ∞  n
X 100
n=1
101

2. ∞  n
X 101
n=1
100

3. ∞
X n
n=1
n+1

4. ∞
X 1+n
n=1
4n − 3

5. ∞  n
X 1

n=1
2

6. ∞ 
X √ 1−n
3
n=1

Question 12.3 Suppose you are given the following options: (i) $13 000 paid
after ten years, or (ii) $1000 paid each year for ten years, first payment is
today. Which of these alternatives would you choose, if the annual interest
rate is 6% per year for the whole period?

24
Question 12.4 Consider the problem

max f (x, y) = x2 y subject to g(x, y) = 2x2 + y 2 = 3.


x,y

Answer the following questions.

1. Set up the Lagrangian for this constrained maximization problem.

2. Find all the solution candidates by using the Lagrangian method.

3. Classify each solution candidate you found as either a local maximum


point or local minimum point.

4. Show that the constrained maximization problem has a solution.

5. Find a solution to the constrained maximization problem.

Question 12.5 Consider the following constrained maximization problem:

max x subject to y − (1 − x)3 ≤ 0, x ≥ 0, y ≥ 0.


(x,y)∈R2

Answer the following questions.

1. Write down the Kuhn-Tucker conditions for the constrained maximiza-


tion problem.

2. Find all the solution candidates satisfying the Kuhn-Tucker conditions.

3. Find a solution to the constrained maximization problem without using


the Kuhn-Tucker conditions.

4. Show that the Kuhn-Tucker conditions fail to generate the solution to the
constrained maximization problem you found in the previous question.

5. Argue why the Kuhn-Tucker conditions fail to generate a solution to the


constrained maximization problem.

Question 12.6 Consider the following constrained maximization problem:


 2
1 1
max 2 − x + − y 2 subject to e−x − y ≤ 0 and y ≤ 23
(x,y)∈R 2 2
Answer the following questions.

1. Write down the Kuhn-Tucker conditions for the constrained maximiza-


tion problem.

25
2. Recall that (x, y) is said to be a feasible point if both e−x − y ≤ 0 and
y ≤ 32 are satisfied. Show that if (x, y) is a feasible point, we must satisfy
x > 0.

3. Define h(x) = e−2x − 2x − 1. Show that h(0) = 0. Show also that h(x)
is a strictly decreasing function for any x > 0.

4. Find all the solution candidates satisfying the Kuhn-Tucker conditions.

5. Find a solution to the constrained maximization problem.

26

You might also like